Difference between revisions of "2019 AMC 10B Problems/Problem 24"

(Solution 2)
Line 51: Line 51:
 
</cmath>
 
</cmath>
 
Choose <math>\boxed{C}</math>
 
Choose <math>\boxed{C}</math>
 
== Solution 2 ==
 
 
Making the reasonable assumption that <math>x_n</math> approaches <math>4</math>, we can translate <math>x</math> down by <math>4</math> to obtain a more simple sequence <math>a_n=x_n-4</math> that should approach  <math>0</math>.
 
 
 
Substitution of <math>(a_{n}+4)</math> for <math>(x_{n})</math> and <math>(a_{n+1}+4)</math> for <math>(x_{n+1})</math> in the definition of <math>x_{n+1}</math> leads to
 
 
 
<math>a_{n+1}+4 = \frac{(a_{n} + 8)(a_{n}+5)}{a_{n}+10} =  \frac{a_{n}^2 + 13a_{n}+40}{a_{n}+10} \implies a_{n+1} = a_{n}(\frac{a_{n}+9}{a_{n}+10}) \implies \frac{a_{n+1}}{a_{n}} = \frac{a_{n}+9}{a_{n}+10}</math>
 
 
 
The ratio of consecutive terms is thus always positive and less than 1 (because <math>a_0</math> is positive). This means that the largest possible value for <math>a_n</math> is 1 and that no value of <math>a_n</math> can be less than or equal to 0.
 
 
 
Plugging the extrema of <math>a_n</math> back into the ratio shows that <math>\frac{9}{10}<\frac{a_{n+1}}{a_{n}}\leq\frac{10}{11}</math> for all <math>n</math>.
 
 
 
For <math>(n>0)</math>, we can bound <math>a_{n}</math> by applying this rule recursively  :  <math>(\frac{9}{10})^n < a_{n} \leq (\frac{10}{11})^n</math> 
 
 
 
 
Therefore, <math>a_{n}</math> is always less than <math>(\frac{1}{2^{20}})</math> when <math>(\frac{10}{11})^n<\frac{1}{2^{20}}\implies n>20log_{\frac{11}{10}}{2}</math>
 
 
and <math>a_{n}</math> is never less than <math>(\frac{1}{2^{20}})</math> when <math>(\frac{9}{10})^n>\frac{1}{2^{20}}\implies n<20log_{\frac{10}{9}}{2}</math>
 
 
 
 
The first integer <math>m</math> such that <math>a_{m}\leq\frac{1}{2^{20}}</math> must therefore lie in the interval
 
<math>\left[\left\lfloor 20log_{\frac{10}{9}}{2}\right\rfloor+1, \left\lceil 20log_{\frac{11}{10}}{2}\right\rceil\right]</math>
 
 
Both of these can be quickly estimated at c. <math>140</math>, so the answer must be <math>\boxed{C}</math>.
 
(actual values are <math>132</math> and <math>147</math>)
 
  
 
==See Also==
 
==See Also==

Revision as of 15:33, 16 February 2019

The following problem is from both the 2019 AMC 10B #24 and 2019 AMC 12B #22, so both problems redirect to this page.

Problem

Define a sequence recursively by $x_0=5$ and \[x_{n+1}=\frac{x_n^2+5x_n+4}{x_n+6}\] for all nonnegative integers $n.$ Let $m$ be the least positive integer such that \[x_m\leq 4+\frac{1}{2^{20}}.\]In which of the following intervals does $m$ lie?

$\textbf{(A) } [9,26] \qquad\textbf{(B) } [27,80] \qquad\textbf{(C) } [81,242]\qquad\textbf{(D) } [243,728] \qquad\textbf{(E) } [729,\infty)$

Solution 1

We first prove that $x_n > 4$ for all $n \ge 0$ by induction from \[x_{n+1} - 4 = \frac{x_n^2 + 5x_n + 4 - 4(x_n+6)}{x_n+6} = \frac{(x_n - 4)(x_n+5)}{x_n+6}\] and then prove $x_n$'s are decreasing by \[x_{n+1} - x_n = \frac{x_n^2 + 5x_n + 4 - x_n(x_n+6)}{x_n+6} = \frac{4-x_n}{x_n+6} < 0\] Now we need to estimate the value of $x_{n+1}-4$ by \[x_{n+1} - 4 = (x_n-4)\cdot\frac{x_n + 5}{x_n+6}\] since $x_n$'s are decreasing, $\frac{x_n + 5}{x_n+6}$ are also decreasing, so we have \[\frac{9}{10} < \frac{x_n + 5}{x_n+6} \le \frac{10}{11}\] and \[\frac{9}{10}(x_n-4) < x_{n+1} - 4 \le \frac{10}{11}(x_n-4)\] which leads to \[(\frac{9}{10})^n = (\frac{9}{10})^n (x_0-4) < x_{n} - 4 \le (\frac{10}{11})^n (x_0-4) = (\frac{10}{11})^n\] The problem requires us to find the value of $n$ such that \[(\frac{9}{10})^n < x_{n} - 4 \le \frac{1}{2^{20}} \text{  and  }  (\frac{10}{11})^{n-1} > x_{n-1} - 4 > \frac{1}{2^{20}}\] using natural logarithm, we need $n \ln \frac{9}{10} < -20 \ln 2$ and $(n-1)\ln \frac{10}{11} > -20 \ln 2$, or

\[n > \frac{20\ln 2}{\ln\frac{10}{9}} \text{  and  }  n-1 < \frac{20\ln 2}{\ln\frac{11}{10}}\]

As estimations, $\ln\frac{10}{9} \approx 1/9$ and $\ln\frac{11}{10} \approx 1/10$, $\ln 2\approx 0.7$ we can estimate that \[126 < n < 141\] Choose $\boxed{C}$

See Also

2019 AMC 10B (ProblemsAnswer KeyResources)
Preceded by
Problem 23
Followed by
Problem 25
1 2 3 4 5 6 7 8 9 10 11 12 13 14 15 16 17 18 19 20 21 22 23 24 25
All AMC 10 Problems and Solutions
2019 AMC 12B (ProblemsAnswer KeyResources)
Preceded by
Problem 21
Followed by
Problem 23
1 2 3 4 5 6 7 8 9 10 11 12 13 14 15 16 17 18 19 20 21 22 23 24 25
All AMC 12 Problems and Solutions

The problems on this page are copyrighted by the Mathematical Association of America's American Mathematics Competitions. AMC logo.png

SUB2PEWDS